Bounded Monotonic Sequence Theorem

Click For Summary
The discussion centers on using the Bounded Monotonic Sequence Theorem to prove the convergence of the sequence defined by {a_i} = {i - √(i² + 1)}. The sequence is shown to be monotonic increasing and has an upper bound, but the challenge lies in establishing a lower bound. It is clarified that for positive integers, the function values are negative and approach zero as i increases, indicating that the sequence does indeed have a lower bound. Ultimately, the conclusion is reached that the sequence is convergent based on these properties.
Euler2718
Messages
90
Reaction score
3

Homework Statement


[/B]
Use the Bounded Monotonic Sequence Theorem to prove that the sequence:

\{a_{i} \} = \Big\{ i - \sqrt{i^{2}+1} \Big\}

Is convergent.

Homework Equations

The Attempt at a Solution


[/B]
I've shown that it has an upper bound and is monotonic increasing, however it is to my understanding that for me to use this theorem the sequence must be bounded (and of course have monotonicity) - i.e. have an upper and lower bound. I can't seem to show that this has a lower bound; even graphically, it just blows off to negative infinity. How should I proceed?
 
Physics news on Phys.org
Morgan Chafe said:

Homework Statement


[/B]
Use the Bounded Monotonic Sequence Theorem to prove that the sequence:

\{a_{i} \} = \Big\{ i - \sqrt{i^{2}+1} \Big\}

Is convergent.

Homework Equations

The Attempt at a Solution


[/B]
I've shown that it has an upper bound and is monotonic increasing, however it is to my understanding that for me to use this theorem the sequence must be bounded (and of course have monotonicity) - i.e. have an upper and lower bound. I can't seem to show that this has a lower bound; even graphically, it just blows off to negative infinity.
Why do you think this? You didn't say anything about the possible values of i, but I assume they are positive integers {1, 2, 3, ...}.

Consider ##f(x) = x - \sqrt{x^2 + 1}##. All of the values of this function are negative, since ##x < \sqrt{x^2 + 1}## for all x > 0. As x gets larger, the difference is smaller, and approaches zero. The largest difference comes when x is smallest (i.e., closest to zero).
Morgan Chafe said:
How should I proceed?
 
  • Like
Likes Euler2718
Morgan Chafe said:

Homework Statement


[/B]
Use the Bounded Monotonic Sequence Theorem to prove that the sequence:

\{a_{i} \} = \Big\{ i - \sqrt{i^{2}+1} \Big\}

Is convergent.

Homework Equations

The Attempt at a Solution


[/B]
I've shown that it has an upper bound and is monotonic increasing, however it is to my understanding that for me to use this theorem the sequence must be bounded (and of course have monotonicity) - i.e. have an upper and lower bound. I can't seem to show that this has a lower bound; even graphically, it just blows off to negative infinity. How should I proceed?

If ##f(i) = i -\sqrt{i^2+1}## is monotonically increasing, then ##f(1) < f(2) < f(3) < \cdot##, so if ##f(1)## is a finite number, it is a lower bound!
 
  • Like
Likes Euler2718
Mark44 said:
Why do you think this? You didn't say anything about the possible values of i, but I assume they are positive integers {1, 2, 3, ...}.

Consider ##f(x) = x - \sqrt{x^2 + 1}##. All of the values of this function are negative, since ##x < \sqrt{x^2 + 1}## for all x > 0. As x gets larger, the difference is smaller, and approaches zero. The largest difference comes when x is smallest (i.e., closest to zero).
Ray Vickson said:
If ##f(i) = i -\sqrt{i^2+1}## is monotonically increasing, then ##f(1) < f(2) < f(3) < \cdot##, so if ##f(1)## is a finite number, it is a lower bound!
Thank you, I think I got it now.
 
Question: A clock's minute hand has length 4 and its hour hand has length 3. What is the distance between the tips at the moment when it is increasing most rapidly?(Putnam Exam Question) Answer: Making assumption that both the hands moves at constant angular velocities, the answer is ## \sqrt{7} .## But don't you think this assumption is somewhat doubtful and wrong?

Similar threads

  • · Replies 7 ·
Replies
7
Views
2K
Replies
15
Views
2K
  • · Replies 9 ·
Replies
9
Views
2K
Replies
6
Views
3K
Replies
3
Views
2K
  • · Replies 5 ·
Replies
5
Views
2K
  • · Replies 13 ·
Replies
13
Views
2K
  • · Replies 1 ·
Replies
1
Views
3K
  • · Replies 2 ·
Replies
2
Views
2K
Replies
7
Views
3K